Next Article in Journal
A Classification of Symmetric (1, 1)-Coherent Pairs of Linear Functionals
Next Article in Special Issue
Truncated Fubini Polynomials
Previous Article in Journal
Stability Analysis of Singly Diagonally Implicit Block Backward Differentiation Formulas for Stiff Ordinary Differential Equations
Previous Article in Special Issue
Ground State Solutions for Fractional Choquard Equations with Potential Vanishing at Infinity
 
 
Font Type:
Arial Georgia Verdana
Font Size:
Aa Aa Aa
Line Spacing:
Column Width:
Background:
Article

On Positive Quadratic Hyponormality of a Unilateral Weighted Shift with Recursively Generated by Five Weights

1
Department of Mathematics, Northeastern University, Shenyang 110-004, China
2
Department of Mathematics, Hannam University, Daejeon 34430, Korea
*
Author to whom correspondence should be addressed.
Mathematics 2019, 7(2), 212; https://doi.org/10.3390/math7020212
Submission received: 16 January 2019 / Revised: 7 February 2019 / Accepted: 8 February 2019 / Published: 25 February 2019
(This article belongs to the Special Issue Polynomials: Theory and Applications)

Abstract

:
Let 1 < a < b < c < d and α ^ 5 : = 1 , a , b , c , d be a weighted sequence that is recursively generated by five weights 1 , a , b , c , d . In this paper, we give sufficient conditions for the positive quadratic hyponormalities of W α x and W α y , x , with α x : x , α ^ 5 and α y , x : y , x , α ^ 5 .

1. Introduction

Let H be a separable, infinite dimensional, complex Hilbert space, and let L ( H ) be the algebra of all bounded linear operators on H . An operator T in L ( H ) is said to be normal if T * T = T T * , hyponormal if T * T T T * , and subnormal if T = N H , where N is normal on some Hilbert space K H . For A , B L ( H ) , let [ A , B ] : = A B B A . We say that an n-tuple T = ( T 1 , , T n ) of operators in L ( H ) is hyponormal if the operator matrix ( [ T j * , T i ] ) i , j = 1 n is positive on the direct sum of n copies of H . For arbitrary positive integer k, T L ( H ) is (strongly) k-hyponormal if ( I , T , , T k ) is hyponormal. It is well known that T is subnormal if and only if T is -hyponormal. An operator T in L ( H ) is said to be weaklyn-hyponormal if p ( T ) is hyponormal for any polynomial p with degree less than or equal to n . An operator T is polynomially hyponormal if p ( T ) is hyponormal for every polynomial p . In particular, the weak two-hyponormality (or weak three-hyponormality) is referred to as quadratical hyponormality (or cubical hyponormality, resp.) and has been considered in detail in [1,2,3,4,5,6,7,8,9].
Let e n n = 0 be the canonical orthonormal basis for Hilbert space l 2 Z + , and let α : = { α n } n = 0 be a bounded sequence of positive numbers. Let W α be a unilateral weighted shift defined by W α e n : = α n e n + 1 n 0 . It is well known that W α is hyponormal if and only if α n α n + 1 n 0 . The moments of W α are usually defined by γ 0 : = 1 , γ i : = α 0 2 α i 1 2 i 1 . It is well known that W α is subnormal if and only if there exists a Borel probability measure μ supported in 0 , W α 2 , with W α 2 supp μ , such that [10] γ n = t n d μ t n 0 . It follows from [11] (Theorem 4) that W α is subnormal if and only if for every k 1 and every n 0 , the Hankel matrix:
A n , k : = γ n γ n + 1 γ n + 2 γ n + k γ n + 1 γ n + 2 γ n + 3 γ n + k + 1 γ n + 2 γ n + 3 γ n + 4 γ n + k + 2 γ n + k γ n + k + 1 γ n + k + 2 γ n + 2 k 0 .
A weighted shift W α is said to be recursively generated if there exists i 1 and φ = φ 0 , , φ i 1 C i such that:
γ n = φ i 1 γ n 1 + + φ 0 γ n i n i ,
where γ n is the moment of W α , i.e., γ 0 : = 1 , γ i : = α 0 2 α i 1 2 i 1 , equivalently,
α n 2 = φ i 1 + φ i 2 α n 1 2 + + φ 0 α n 1 2 α n i + 1 2 n i .
Given an initial segment of weights α : α 0 , , α 2 k k 0 , there is a canonical procedure to generate a sequence (denoted α ^ ) in such a way that W α ^ is a recursively-generated shift having α as an initial segment of weight. In particular, given an initial segment of weights α : a , b , c with 0 < a < b < c , we obtain φ 0 = a b c b b a and φ 1 = b c a b a .
In [12,13], Curto-Putinar proved that there exists an operator that is polynomially hyponormal, but not two-hyponormal. Although the existence of a weighted shift, which is polynomially hyponormal, but not subnormal, was established in [12,13], a concrete example of such weighted shifts has not been found yet. Recently, the authors in [14] proved that the subnormality is equivalent to the polynomial hyponormality for recursively-weighted shift W α with α : x , a , b , c . Based on this, in this paper, we have to consider the weighted shift operator with five generated elements.
The organization of this paper is as follows. In Section 2, we recall some terminology and notations concerning the quadratic hyponormality and positive quadratic hyponormality of unilateral weighted shifts W α . In Section 3, we give some results on the unilateral weighted shifts with recursively generated by five weights α ^ 5 : = 1 , a , b , c , d 1 < a < b < c < d . In Section 4, we consider positive quadratic hyponormalities of W α with weights α : x , α ^ 5 and α : y , x , α ^ 5 . In Section 5, we give more results on the positive quadratic hyponormality for any unilateral weighted shift W α . In Section 6, we present the conclusions.

2. Preliminaries and Notations

Recall that a weighted shift W α is quadratically hyponormal if W α + s W α 2 is hyponormal for any s C [2], i.e., D ( s ) : = [ ( W α + s W α 2 ) * , W α + s W α 2 ] 0 , for any s C . Let { e i } i = 0 be an orthonormal basis for H , and let P n be the orthogonal projection on i = 0 n { e i } . For s C , we let:
D n ( s ) = P n [ ( W α + s W α 2 ) * , W α + s W α 2 ] P n = q 0 r 0 0 0 0 r 0 ¯ q 1 r 1 0 0 0 r 1 ¯ q 2 0 0 0 0 0 q n 1 r n 1 0 0 0 r n 1 ¯ q n ,
where:
q k : = u k + s 2 v k , r k : = w k s ¯ , u k : = α k 2 α k 1 2 , v k : = α k 2 α k + 1 2 α k 2 2 α k 1 2 , w k : = α k 2 ( α k + 1 2 α k 1 2 ) 2 , for k 0 ,
and α 1 = α 2 : = 0 . Hence, W α is quadratically hyponormal if and only if D n ( s ) 0 for every s C and every n 0 . Hence, we consider d n · : = det D n · , which is a polynomial in t : = s 2 of degree n + 1 , with Maclaurin expansion d n t : = i = 0 n + 1 c n , i t i . It is easy to find the following recursive relations [2]:
d 0 t = q 0 , d 1 t = q 0 q 1 r 0 2 , d n + 2 t = q n + 2 d n + 1 t r n + 1 2 d n t n 0 .
Furthermore, we can obtain the following:
c 0 , 0 = u 0 , c 0 , 1 = v 0 , c 1 , 0 = u 1 u 0 , c 1 , 1 = u 1 v 0 + u 0 v 1 w 0 , c 1 , 2 = v 1 v 0 ,
and:
c n + 2 , i = u n + 2 c n + 1 , i + v n + 2 c n + 1 , i 1 w n + 1 c n , i 1 n 0 , and 0 i n + 1 .
In particular, for any n 0 , we have:
c n , 0 = u 0 u 1 u n , c n , n + 1 = v 0 v 1 v n .
Furthermore, we can obtain the following results.
Lemma 1.
Let ρ : = v 2 u 0 v 1 w 0 + v 0 u 1 v 2 w 1 . Then, for any n 4 , we have:
c n , n = u n c n 1 , n + u n 1 v n w n 1 c n 2 , n 1 + i = 1 n 3 v n v n 1 v i + 3 u i + 1 v i + 2 w i + 1 c i , i + 1 + v n v n 1 v 3 ρ .
Lemma 2.
Let τ : = u 0 u 1 v 2 w 1 . Then, for any n 4 , we have:
c n , n 1 = u n c n 1 , n 1 + u n 1 v n w n 1 c n 2 , n 2 + i = 1 n 3 v n v n 1 v i + 3 u i + 1 v i + 2 w i + 1 c i , i + v n v n 1 v 3 τ .
Lemma 3.
For any n 5 and 0 i n 2 , we have:
c n , i = u n c n 1 , i + u n 1 v n w n 1 c n 2 , i 1 + j = 1 n 3 v n v n 1 v j + 3 u j + 1 v j + 2 w j + 1 c j , j + i n + 1 + v n v n 1 v 5 c i n + 5 , 0 u i n + 6 v i n + 7 w i n + 6 .
To detect the positivity of d n t , we need the following concept.
Definition 1.
Let α : α 0 , α 1 , be a positive weight sequence. We say that W α is positively quadratically hyponormal if c n , i 0 for all n , i 0 with 0 i n + 1 , and c n , n + 1 > 0 for all n 0 [2].
Positive quadratic hyponormality implies quadratic hyponormality, but the converse is false [15]. In addition, the authors in [15] showed that the positive quadratic hyponormality is equivalent to the quadratic hyponormality for recursively-generated weighted shift W α with α : x , a , b , c (here, 0 < x a < b < c ).

3. Recursive Relation of W α ^ 5

Given the initial segment of weights α : 1 , a , b , c , d with 1 < a < b < c < d , we obtain the moments:
γ 0 = γ 1 = 1 , γ 2 = a , γ 3 = a b , γ 4 = a b c , γ 5 = a b c d .
Let:
V 0 = γ 0 γ 1 γ 2 , V 1 = γ 1 γ 2 γ 3 , V 2 = γ 2 γ 3 γ 4 ,
and we assume that V 0 , V 1 , V 2 are linearly independent, i.e.,
det V 0 , V 1 , V 2 = det A ( 0 , 2 ) 0 .
Then, there exist three nonzero numbers φ 0 , φ 1 , φ 2 , such that:
γ 3 γ 4 γ 5 = φ 0 γ 0 γ 1 γ 2 + φ 1 γ 1 γ 2 γ 3 + φ 2 γ 2 γ 3 γ 4 .
A straightforward calculation shows that:
φ 0 = a b ( a b 2 2 a b c + b c 2 + a c d b c d ) a 2 2 a b + a b 2 + b c a b c , φ 1 = a b ( a b a c b c + b c 2 + c d b c d ) a 2 2 a b + a b 2 + b c a b c , φ 2 = b ( a 2 a b a c + a b c + c d a c d ) a 2 2 a b + a b 2 + b c a b c .
Thus:
γ n + 1 = φ 0 γ n 2 + φ 1 γ n 1 + φ 2 γ n n 5 ,
i.e.,
α n 2 = φ 2 + φ 1 α n 1 2 + φ 0 α n 1 2 α n 2 2 n 5 .
By (1), we can obtain a recursively-generated weighted shift, and we set it as α ^ 5 . In this case, we call the weighted shift operator W α ^ 5 with rank three.
Proposition 1.
W α ^ 5 with rank three is subnormal if and only if:
(1) 1 < a < b ,
(2) c > a b b 1 2 a 1 + 1 ,
(3) d > b c c a 2 b a + a .
Proof. 
See [16], Example 3.6. □
Proposition 2.
If W α ^ 5 with rank three is subnormal, then φ 0 > 0 , φ 1 < 0 and φ 2 > 0 .
Proof. 
By Proposition 1, we know that:
ϝ 1 : = a b 2 2 a b + b c + a 2 a b c < 0 , ϝ 2 : = a b 2 + b c 2 2 a b c + a c d b c d < 0 .
Thus, φ 0 > 0 . Since:
a b a c b c + b c 2 + c 1 b d < c b ϝ 1 b a < 0 ,
and:
a 2 a b a c + a b c + c 1 a d < c a ϝ 1 b a < 0 ,
we have φ 1 < 0 and φ 2 > 0 . The proof is complete. □
Proposition 3.
Let u n 1 u n = β n n 5 . Then:
β n = α n 1 2 α n 2 2 α n 3 2 φ 1 α n 3 2 + φ 0 + φ 0 β n 1 .
Proof. 
Since:
u n = α n 2 α n 1 2 = φ 1 α n 1 2 α n 2 2 u n 1 φ 0 α n 1 2 α n 2 2 α n 3 2 u n 1 + u n 2 = φ 1 α n 1 2 α n 2 2 + φ 0 α n 1 2 α n 2 2 α n 3 2 u n 1 φ 0 α n 1 2 α n 2 2 α n 3 2 u n 2 ,
so:
1 = φ 1 α n 1 2 α n 2 2 + φ 0 α n 1 2 α n 2 2 α n 3 2 u n 1 u n φ 0 α n 1 2 α n 2 2 α n 3 2 u n 2 u n = φ 1 α n 1 2 α n 2 2 + φ 0 α n 1 2 α n 2 2 α n 3 2 + φ 0 α n 1 2 α n 2 2 α n 3 2 u n 2 u n 1 u n 1 u n .
Thus, we have:
β n = 1 φ 1 α n 1 2 α n 2 2 + φ 0 α n 1 2 α n 2 2 α n 3 2 + φ 0 α n 1 2 α n 2 2 α n 3 2 β n 1 = α n 1 2 α n 2 2 α n 3 2 φ 1 α n 3 2 + φ 0 + φ 0 β n 1 .
Thus, we have our conclusion. □
Since lim n α n 2 = L 2 , we let lim n β n : = β * , and by (2), we have β * = L 6 φ 1 L 2 + φ 0 + φ 0 β * ; hence:
β * = 1 2 φ 1 2 φ 0 L 2 + 1 2 φ 0 φ 1 2 4 φ 0 φ 2 L 4 2 φ 0 φ 1 L 2 3 φ 0 2 .

4. Main Results

First, we give the following result (cf. [11], Corollary 5).
Proposition 4.
Let W α be any unilateral weighted shift. Then, W α is two-hyponormal if and only if θ k : = u k v k + 1 w k 0 , k N .
It is well-known that if W α is two-hyponormal or positively quadratically hyponormal, then W α is quadratically hyponormal. By Proposition 4 and Lemma 1∼3, we have the following result.
Theorem 1.
Let W α be any unilateral weighted shift. If W α is 2-hyponormal, then W α is positively quadratically hyponormal.

4.1. The Positive Quadratic Hyponormality of W α x

Let α x : x , α ^ 5 with 0 < x 1 , and we consider the following n + 1 × n + 1 matrix:
D n = u 0 + v 0 t w 0 t 0 0 w 0 t u 1 + v 1 t w 1 t 0 0 w 1 t u 2 + v 2 t 0 w n 1 t 0 0 w n 1 t u n + v n t .
Let d n = det D n = i = 0 n + 1 c n , i t i . Then:
Lemma 4.
c ( n , i ) 0 , n = 0 , 1 , 2 , and 0 i n + 1 .
Proof. 
In fact, c ( 1 , 1 ) = x a x > 0 , c ( 2 , 1 ) = a x 1 x b 1 > 0 , and:
c ( 2 , 2 ) = a x a b 1 b 1 x > a x a b a b 1 x = a x a x b 1 > 0 .
Thus, we have our conclusion. □
Lemma 5.
Assume that θ k : = u k v k + 1 w k 0 for k 2 . Then, for n 3 , 0 i n + 1 , we have:
c ( n , i ) u n c ( n 1 , i ) + v n v 3 [ v 2 c ( 1 , i n + 1 ) w 1 c ( 0 , i n + 1 ) ] .
Proof. 
For n = 3 , 0 i 4 ,
c ( 3 , i ) = u 3 c ( 2 , i ) + v 3 c ( 2 , i 1 ) w 2 c ( 1 , i 1 ) = u 3 c ( 2 , i ) + ( u 2 v 3 w 2 ) c ( 1 , i 1 ) + v 3 [ v 2 c ( 1 , i 2 ) w 1 c ( 0 , i 2 ) ] u 3 c ( 2 , i ) + v 3 [ v 2 c ( 1 , i 2 ) w 1 c ( 0 , i 2 ) ] .
By the inductive hypothesis, we have our result. □
Thus, if θ k : = u k v k + 1 w k 0 for k 2 , then by Lemma 1∼3 and Lemma 5, for n 3 , we have:
c ( n , i ) v n v 2 c ( 1 , 2 ) , for i = n + 1 , u n c ( n 1 , n ) + v n v 3 ρ , for i = n , u n c ( n 1 , n 1 ) + v n v 3 τ , for i = n 1 , u n c ( n 1 , i ) , for 0 i n 2 ,
where ρ = a x b 1 a x > 0 , τ = x a b a a b 2 a + 1 x . Therefore, when n 3 , we have:
c ( n , n + 1 ) > 0 , c ( n , n ) > u n c ( n 1 , n ) + v n v 3 ρ 0 , c ( n , i ) u n u i + 2 c ( i + 1 , i ) ( n 3 , 0 i n 2 ) .
To complete our analysis of the coefficients c ( n , i ) , it suffices to determine the values of x for which c ( n , n 1 ) 0 ( n 3 ) .
Lemma 6.
c ( n , n 1 ) 0 (for any n 3 ) , if c ( 3 , 2 ) 0 , c ( 4 , 3 ) 0 and A n : = v 0 v 1 v 2 u n u n 1 + u n v n 1 ρ + v n v n 1 τ 0 ( n 5 ) .
Proof. 
For n 4 , by Lemma 2, we have:
c ( n , n 1 ) u n c ( n 1 , n 1 ) + v n v 3 τ u n [ u n 1 c ( n 2 , n 1 ) + v n 1 v 3 ρ ] + v n v 3 τ ,
and since c ( n 2 , n 1 ) = v n 2 v 0 , we get:
c ( n , n 1 ) u n ( u n 1 v n 2 v 0 + v n 1 v 3 ρ ) + v n v 3 τ .
If n 5 , we can factor v n 2 v 3 to get:
c ( n , n 1 ) v n 2 v 3 ( v 0 v 1 v 2 u n u n 1 + u n v n 1 ρ + v n v n 1 τ ) = v n 2 v 3 A n .
Hence, we have our result. □
Let:
x n : = sup { x : c ( n , n 1 ) 0 in W α } ( n 3 ) .
By direct computations, we have:
x 3 = a θ 2 + a b a v 3 + a a b 1 u 3 θ 2 + a b 1 u 3 + 2 a + a b + 1 v 3 , x 4 = a a b 1 θ 3 + a u 4 + v 4 θ 2 + a 2 b 1 u 4 + v 4 a b a v 3 + a 2 b u 3 u 4 a b 1 θ 3 + v 4 θ 2 + a b 2 a + 1 v 4 + u 4 a b 1 v 3 + a u 3 u 4 .
For n 5 , a calculation using the specific form of v 0 , v 1 , v 2 , ρ and τ shows that:
A n = [ a 2 b u n u n 1 + a 2 ( b 1 ) u n v n 1 + a ( b a ) v n v n 1 a u n u n 1 + a ( b 1 ) u n v n 1 + ( a b + 1 2 a ) v n v n 1 x ] x ,
it follows that:
x n = a 2 b u n u n 1 + a 2 ( b 1 ) u n v n 1 + a ( b a ) v n v n 1 a u n u n 1 + a ( b 1 ) u n v n 1 + ( a b + 1 2 a ) v n v n 1 .
Let z n : = v n u n ( u n 0 ) . Then, for n 5 ,
x n = a 2 b + a 2 ( b 1 ) z n 1 + a ( b a ) z n z n 1 a + a ( b 1 ) z n 1 + ( a b + 1 2 a ) z n z n 1 .
Lemma 7.
lim n z n = K : = ( 1 + β * ) φ 2 φ 0 L 4 β * , where β * as in (3).
Proof. 
Since:
α n + 1 2 = φ 2 + φ 1 α n 2 + φ 0 α n 2 α n 1 2 ( n 5 ) ,
from which it follows that:
α n 2 α n + 1 2 = φ 2 α n 2 + φ 1 + φ 0 α n 1 2 .
Thus:
v n = φ 2 ( u n + u n 1 ) φ 0 ( u n 1 + u n 2 ) α n 1 2 α n 3 2 . n 5
Thus, we have (for n 5 ):
z n = φ 2 ( 1 + β n ) φ 0 α n 1 2 α n 3 2 β n ( 1 + β n 1 ) .
Since α n 2 L 2 , we have:
lim n z n = ( 1 + β * ) φ 2 φ 0 L 4 β * .
Thus, we have our conclusion. □
Let:
f ( z , w ) : = a 2 b + a 2 ( b 1 ) z + a ( b a ) z w a + a ( b 1 ) z + ( a b + 1 2 a ) z w .
By Lemma 7 and the fact in [2] (p. 399), if z n is increasing, then we know that x n n 5 in (5) is decreasing and inf n 5 x n = f ( K , K ) . Thus, we have the following result.
Theorem 2.
Assume that W α ^ 5 with rank three is subnormal. Let α x : x , α ^ 5 , and let:
h 2 + : = sup { x : W α x be positively quadratically hyponormal } .
If z n : = v n u n n 5 is increasing, then:
h 2 + min 1 , x 3 , x 4 , a 2 b + a 2 ( b 1 ) K + a ( b a ) K 2 a + a ( b 1 ) K + ( a b + 1 2 a ) K 2 ,
where:
x 3 = a θ 2 + a b a v 3 + a a b 1 u 3 θ 2 + a b 1 u 3 + 2 a + a b + 1 v 3 , x 4 = a a b 1 θ 3 + a u 4 + v 4 θ 2 + a 2 b 1 u 4 + a b a v 4 v 3 + a 2 b u 3 u 4 a b 1 θ 3 + v 4 θ 2 + a b 1 u 4 + a b 2 a + 1 v 4 v 3 + a u 3 u 4 , K = ( 1 + β * ) φ 2 φ 0 L 4 β * .
Remark 1.
By (6), we know that if β n is increasing, then so is z n . Hence, our problems are as follows.
Problem 1.
Let α : = { α n } n = 0 be any unilateral weighted sequence. If W α is subnormal, is β n increasing or not? In particular, what is the answer for subnormal W α ^ 5 with rank three?
Example 1.
Let α x : x , 1 , 2 , 3 , 4 , 5 . Then, φ 0 = 6 , φ 1 = 18 , φ 2 = 9 , L 2 6 . 2899 , and K 32 . 118 , x 3 = 17 18 0 . 94444 , x 4 = 226 249 0 . 90763 , f ( K , K ) 0 . 77512 . We obtain h 2 + 0 . 77512 . That is, if 0 < x 0 . 77512 , then W α x is positively quadratically hyponormal. Numerically, we know that β n and z n are all increasing. See the following Table 1.

4.2. The Positive Quadratic Hyponormality of W α y , x

Let α y , x : y , x , α ^ 5 . We also consider the matrix as in (4), and let d n = det D n = i = 0 n + 1 c n , i t i . Then:
c 1 , 1 = x y 1 y > 0 , c 2 , 1 = y x y a x > 0 , c 2 , 2 = x y a a y + x y x 2 x y 1 y a x > 0 ,
and:
c 3 , 1 u 3 c ( 2 , 1 ) > 0 , c 3 , 2 = y x a x 2 2 x + a b y + x 1 a x 2 + a 1 b x + a a b 1 , c 3 , 3 u 3 c ( 2 , 3 ) + v 3 ρ > 0 .
Since:
ρ = x y 1 y a x > 0 , τ = y 2 x x 2 a y + x a 1 ,
we can similarly show that c ( n , n 1 ) 0 for all n 3 , if c ( 3 , 2 ) 0 , c ( 4 , 3 ) 0 and y x a 1 K 2 + a x K + a x a 2 x + x 2 K 2 + a x x K + x 3 , where K = ( 1 + β * ) φ 2 φ 0 L 4 β * . Thus, we have the following result.
Theorem 3.
Let α y , x : y , x , α ^ 5 . If z n : = v n u n n 5 is increasing, and:
(1) 0 < x min a a a 1 , a b c 1 a b a 1 a 2 b + b c 2 a b a c + 2 a 2 a 3 a b c a a 1 + b c a ,
(2) 0 < y min x , f 1 x , f 2 x , f 3 x , where:
f 1 x = x a a b 1 + a 1 b x a 1 x 2 a x a b 2 x + x 2 , f 2 x = x p 1 x p 2 x , with : p 1 x = a 2 b + b c a 2 a b c x 2 + a 2 b 2 a b + a 2 a b 2 c + a b c x + c a 2 b 2 2 a 3 b + 2 a 2 b c a b , p 2 x = b c a b a + a 2 x 3 + a a 2 b + 3 a b 2 b c a b c x 2 + a 3 b 3 a 2 b a 2 + c a b 2 + 2 c a b x + a 2 b a b c , f 3 x = x a 1 K 2 + a x K + a x a 2 x + x 2 K 2 + a x x K + x 3 , K = ( 1 + β * ) φ 2 φ 0 L 4 β * ,
then W α y , x is positively quadratically hyponormal.
Example 2.
Let a = 2 , b = 3 , c = 4 , d = 5 . If 0 < x 2 2 0 . 585 79 , y x K 2 + 2 x K + 2 x x 2 2 x + 2 K 2 + 2 x x 2 K + x 3 with K 32 . 118 , then W α y , x is positively quadratically hyponormal. See the following Figure 1.

5. More Results

From the above discussions, we obtain the following criteria for any unilateral weighted shifts.
Proposition 5.
Let α x : x , 1 , a , b , c , d , α 5 , α 6 , , and α : 1 , a , b , c , d , α 5 , α 6 , be subnormal weighted shifts. Let:
h 2 + : = sup { x : W α x be positively quadratically hyponormal } .
If z n = v n u n n 5 is increasing and z n K (as n ), then:
h 2 + min 1 , x 3 , x 4 , a 2 b + a 2 ( b 1 ) K + a ( b a ) K 2 a + a ( b 1 ) K + ( a b + 1 2 a ) K 2 ,
where:
x 3 = a θ 2 + a b a v 3 + a a b 1 u 3 θ 2 + a b 1 u 3 + 2 a + a b + 1 v 3 , x 4 = a a b 1 θ 3 + a u 4 + v 4 θ 2 + a 2 b 1 u 4 + a b a v 4 v 3 + a 2 b u 3 u 4 a b 1 θ 3 + v 4 θ 2 + a b 1 u 4 + a b 2 a + 1 v 4 v 3 + a u 3 u 4 .
By Proposition 5, we can have the following results, but we omit the concrete computations.
Example 3.
(1) Let α x : x , 2 3 , 3 4 , 4 5 , . Then, h 2 + = 2 3 (cf. [11], Proposition 7).
(2) Let α x : x , 5 8 , 3 4 , 4 5 , . Then, h 2 + = 1945 3136 (cf. [15], Theorem 3.7).
(3) Let α x : x , 1 , 2 , 2 . 1 , 12 . 1 . Then, h 2 + 0 . 16682 .
(4) Let α x : x , n n + 1 · 1 2 · 2 n + 1 1 2 n 1 . Then, h 2 + = 3 4 (cf. [16], Example 3.4).

6. Conclusions

In this work, we study a weighed shift operator for which the weights are recursively generated by five weights. We give sufficient conditions of the positive quadratic hyponormalities. Next, it is worth studying the cubic hyponormality, semi-weak k-hyponormalities, and so on.

Author Contributions

All authors contributed equally in writing this article. All authors read and approved the final manuscript.

Funding

This work was supported by the National Research Foundation of Korea (NRF) grant funded by the Korea government (MEST) (No. 2017R1A2B4006092).

Conflicts of Interest

The authors declare no conflict of interest.

References

  1. Baek, S.; Exner, G.; Jung, I.B.; Li, C. Semi-cubic hyponormality of weighted shifts with Stampfli recursive tail. Integral Equ. Oper. Theory 2017, 88, 229–248. [Google Scholar] [CrossRef]
  2. Curto, R.; Fialkow, L. Recursively generated weighted shifts and the subnormal completion problem, II. Integral Equ. Oper. Theory 1994, 18, 369–426. [Google Scholar] [CrossRef]
  3. Curto, R.; Jung, I.B. Quadratically hyponormal weighted shifts with first two equal weights. Integral Equ. Oper. Theory 2000, 37, 208–231. [Google Scholar] [CrossRef]
  4. Dong, Y.; Exner, G.; Jung, I.B.; Li, C. Quadratically hyponormal recursively generated weighted shifts. Oper. Theory Adv. Appl. 2008, 187, 141–155. [Google Scholar]
  5. Exner, G.; Jung, I.B.; Park, S.S. Weakly n-hyponormal weighted shifts and their examples. Integral Equ. Oper. Theory 2006, 54, 215–233. [Google Scholar] [CrossRef]
  6. Li, C.; Lee, M.R. Existence of non-subnormal completely semi-weakly hyponormal weighted shifts. Filomat 2017, 31, 1627–1638. [Google Scholar] [CrossRef]
  7. Jung, I.B.; Park, S.S. Cubically hyponormal weighted shifts and their examples. J. Math. Anal. Appl. 2000, 247, 557–569. [Google Scholar] [CrossRef]
  8. Li, C.; Cho, M.; Lee, M.R. A note on cubically hyponormal weighted shifts. Bull. Korean Math. Soc. 2014, 51, 1031–1040. [Google Scholar] [CrossRef]
  9. Exner, G.; Jin, J.Y.; Jung, I.B.; Lee, J.E. Weak Hamburger-type weighted shifts and their examples. J. Math. Anal. Appl. 2018, 462, 1357–1380. [Google Scholar] [CrossRef]
  10. Curto, R.; Fialkow, L. Recursively generated weighted shifts and the subnormal completion problem. Integral Equ. Oper. Theory 1993, 17, 202–246. [Google Scholar] [CrossRef]
  11. Curto, R. Quadratically hyponormal weighted shifts. Integral Equ. Oper. Theory 1990, 13, 49–66. [Google Scholar] [CrossRef]
  12. Curto, R.; Putinar, M. Existence of non-subnormal polynomially hyponormal operators. Bull. Am. Math. Soc. 1991, 25, 373–378. [Google Scholar] [CrossRef]
  13. Curto, R.; Putinar, M. Nearly subnormal operators and moment problems. J. Funct. Anal. 1993, 115, 480–497. [Google Scholar] [CrossRef]
  14. Li, C.; Lee, M.R.; Baek, S.H. A relationship: Subnormal, polynomially hyponormal and semi-weakly hyponormal weighted shifts. J. Math. Anal. Appl. 2019. under review. [Google Scholar]
  15. Jung, I.B.; Park, S.S. Quadratically hyponormal weighted shifts and their examples. Integral Equ. Oper. Theory 2002, 36, 480–498. [Google Scholar] [CrossRef]
  16. Jung, I.B.; Li, C. A formula for k-hyponormality of backstep extensions of subnormal weighted shifts. Proc. Am. Math. Soc. 2001, 129, 2343–2351. [Google Scholar] [CrossRef]
Figure 1. A subset of the region of positive quadratic hyponormality of W α y , x in Example 2.
Figure 1. A subset of the region of positive quadratic hyponormality of W α y , x in Example 2.
Mathematics 07 00212 g001
Table 1. Numerical data for β n and z n in Example 1.
Table 1. Numerical data for β n and z n in Example 1.
n789101112
β n 1.9851 2.3929 2.6008 2.6882 2.7218 2.7343
z n 25.597 29.120 30.909 31.660 31.949 32.056
n131415161718
β n 2.7389 2.7406 2.7412 2.7414 2.7415 2.7415
z n 32.096 32.11 32.116 32.117 32.118 32.118

Share and Cite

MDPI and ACS Style

Li, C.; Ryoo, C.S. On Positive Quadratic Hyponormality of a Unilateral Weighted Shift with Recursively Generated by Five Weights. Mathematics 2019, 7, 212. https://doi.org/10.3390/math7020212

AMA Style

Li C, Ryoo CS. On Positive Quadratic Hyponormality of a Unilateral Weighted Shift with Recursively Generated by Five Weights. Mathematics. 2019; 7(2):212. https://doi.org/10.3390/math7020212

Chicago/Turabian Style

Li, Chunji, and Cheon Seoung Ryoo. 2019. "On Positive Quadratic Hyponormality of a Unilateral Weighted Shift with Recursively Generated by Five Weights" Mathematics 7, no. 2: 212. https://doi.org/10.3390/math7020212

Note that from the first issue of 2016, this journal uses article numbers instead of page numbers. See further details here.

Article Metrics

Back to TopTop